LSAT and Law School Admissions Forum

Get expert LSAT preparation and law school admissions advice from PowerScore Test Preparation.

 15veries
  • Posts: 113
  • Joined: Sep 25, 2016
|
#30745
Hi

So I was not sure between B and D.

For B, this is actually compatible with the explanation in paragraph 4, so it does not make the explanation in paragraph in 3 more favorble? (so not sure which is better actually)

For D,I was not sure because this seems also compatible with explanation in paragraph 4...because it also talk about relationship with other community members and thus why they show the expression.
Why D correct and B wrong ?
User avatar
 Jonathan Evans
PowerScore Staff
  • PowerScore Staff
  • Posts: 726
  • Joined: Jun 09, 2016
|
#31558
Hi, 15veries,

A "Strengthen" question such as this must be approached in the same manner as one would a Logical Reasoning strengthen question, that is, you must identify the claim, the existing premises, and what kind of support might help the claim. Usually on these LR type reading comp questions, the reasoning itself will be more straightforward than on LR strengthen questions, but students do frequently have trouble with these. Some of the difficulty stems from the fact that students are unaccustomed to dealing with logical reasoning situations on reading comp. Therefore, your first order of business is to break the scenario down into the proper components.

The claim in Paragraph 3 is that the fair divisions common in the Ultimate Game stem from the long-term need to ensure a strong group, one in which all members contribute to each other according to their strengths. "From each according to his ability; to each according to his need" as it were. ;)

The evidence that supports this claim is that it explains why the proposers would make equitable offers.

However, the author of the passage actually makes explicit which evidence is missing or unsatisfactory, i.e. this explanation makes no account of why others would reject low offers (ln 37).

To prephrase, you would need to find evidence that remedies this gap, you need to find a reason consistent with this "strong group" hypothesis that explains why responders reject the low offers.

Answer Choice B is not relevant. In fact, it alludes more to information either in paragraph 2 or paragraph 4. It is out of scope and does not help the claim in paragraph 3.

Answer Choice D provides "supporter" evidence that fills in the gap about responders' rejecting low offers. If it were equally counterproductive to the functioning of small groups for responders to reject low offers, then we have direct evidence that accounts for the information that the author found lacking, thus bolstering the "small group" hypothesis in paragraph 3.
 ataraxia10
  • Posts: 46
  • Joined: Oct 04, 2018
|
#64590
I chose C over D, thinking that if the whole idea of keeping secrets was difficult to carry out, it would make the previous hypothesis in paragraph 3 more favorable in the eyes of the author. Is it possible for reading comp strengthen questions to have an answer choice that allows us to weaken the other idea that is being compared as opposed to strengthen the idea in question? Almost like a seesaw effect? Perhaps I am not making sense. I felt like that was one of the options we had for LR strengthen questions: either weaken the other possibilities or strengthen the idea in question.
 Adam Tyson
PowerScore Staff
  • PowerScore Staff
  • Posts: 5153
  • Joined: Apr 14, 2011
|
#64655
One good reason to reject answer C here, ataraxia10, is that the third paragraph has nothing to do with secrets. It's only about outcompeting rivals. Our author dislikes this explanation because it appears to only explain half of the problem - it tells us why offerors offer large amounts, but not why offerees reject small offers.

This question is about strengthening the explanation in the third paragraph to get the author to like it more. Since his objection is that it fails to explain the rejection of low offers, we need an answer that adds to that hypothesis an explanation for that behavior. You might say we are looking to weaken the author's objection, which I do think is a valid approach here, but it still has to come from making the hypothesis in the third paragraph more palatable, more complete in the author's eyes. It's not about weakening his preferred explanation, because even if we destroy that explanation he will still have objections to the other one. So, focus on improving the hypothesis in the third paragraph, and you can pretty much ignore the explanation in the fourth paragraph.

Once we know that we have to remove the objection, we can look for an answer that makes us say "oh, that's why the offeree rejects the low offer!" within the context of maintaining the support of a strong group and dealing with competition from rivals. Answer D does just that.

Get the most out of your LSAT Prep Plus subscription.

Analyze and track your performance with our Testing and Analytics Package.